Top Posters
Since Sunday
e
5
R
5
e
4
4
d
4
o
3
p
3
t
3
3
m
3
p
3
m
3
New Topic  
jerico jerico
wrote...
Posts: 4603
Rep: 8 0
9 years ago
A segment has the following data:

Sales   $630,000
Variable costs   336,000
Fixed costs   325,500

What will be the incremental effect on net income if this segment is eliminated, assuming the fixed costs will be allocated to profitable segments?
A) $304,500 increase
B) $304,000 decrease
C) $294,000 decrease
D) $325,500 decrease
Textbook 
Cost Accounting

Cost Accounting


Edition: 14th
Authors:
Read 1482 times
5 Replies
Replies
Answer verified by a subject expert
cyborgcyborg
wrote...
Top Poster
Posts: 4566
9 years ago
Sign in or Sign up in seconds to unlock everything for free
More solutions for this book are available here

Related Topics

jerico Author
wrote...
9 years ago
I can confidently say that it looks and sounds right lol Thank you Slight Smile Give this man a thumbs up.
wrote...
9 years ago
Cool! No problem.
wrote...
3 years ago
thank you man
wrote...
2 years ago
Thanks
New Topic      
Explore
Post your homework questions and get free online help from our incredible volunteers
  909 People Browsing
Related Images
  
 651
  
 274
  
 2887
Your Opinion
Who will win the 2024 president election?
Votes: 7
Closes: November 4